Difference between revisions of "1959 AHSME Problems/Problem 7"
(→Solution) |
(→Solution) |
||
Line 1: | Line 1: | ||
==Solution== | ==Solution== | ||
− | If we let <math>a=3</math> and <math>d=1</math>, then we will get a <math>3</math>-<math>4</math>-<math>5</math> triangle, which is a right triangle. So, the answer is <math>\boxed{\textbf{(D)}3:1}</math>. | + | If we let <math>a=3</math> and <math>d=1</math>, then we will get a <math>3</math>-<math>4</math>-<math>5</math> triangle, which is a right triangle. So, the answer is <math>\boxed{\textbf{(D)} \ 3:1}</math>. |
Revision as of 04:56, 24 September 2018
Solution
If we let and , then we will get a -- triangle, which is a right triangle. So, the answer is .